Difference between revisions of "2020 AMC 12B Problems/Problem 19"

(Creation of Page)
 
(Redirected page to 2020 AMC 10B Problems/Problem 23)
(Tag: New redirect)
 
(11 intermediate revisions by 3 users not shown)
Line 1: Line 1:
Square <math>ABCD</math> in the coordinate plane has vertices at the points <math>A(1,1), B(-1,1), C(-1,-1),</math> and <math>D(1,-1).</math> Consider the following four transformations: <math>L,</math> a rotation of <math>90^{\circ}</math> counterclockwise around the origin; <math>R,</math> a rotation of <math>90^{\circ}</math> clockwise around the origin; <math>H,</math> a reflection across the <math>x</math>-axis; and <math>V,</math> a reflection across the <math>y</math>-axis.
+
#REDIRECT [[2020 AMC 10B Problems/Problem 23]]
 
 
Each of these transformations maps the squares onto itself, but the positions of the labeled vertices will change. For example, applying <math>R</math> and then <math>V</math> would send the vertex <math>A</math> at <math>(1,1)</math> to <math>(-1,-1)</math> and would send the vertex <math>B</math> at <math>(-1,1)</math> to itself. How many sequences of <math>20</math> transformations chosen from <math>\{L, R, H, V\}</math> will send all of the labeled vertices back to their original positions? (For example, <math>R, R, V, H</math> is one sequence of <math>4</math> transformations that will send the vertices back to their original positions.)
 
 
 
<math>\textbf{(A)}\ 2^{37} \qquad\textbf{(B)}\ 3\cdot 2^{36} \qquad\textbf{(C)}\  2^{38} \qquad\textbf{(D)}\ 3\cdot 2^{37} \qquad\textbf{(E)}\ 2^{39}</math>
 
 
 
==Solution==
 
 
 
none as of yet...
 
 
 
{{AMC12 box|year=2020|ab=B|num-b=18|num-a=20}}
 
{{MAA Notice}}
 

Latest revision as of 21:22, 12 February 2020